LSAT and Law School Admissions Forum

Get expert LSAT preparation and law school admissions advice from PowerScore Test Preparation.

 Administrator
PowerScore Staff
  • PowerScore Staff
  • Posts: 8919
  • Joined: Feb 02, 2011
|
#24691
Complete Question Explanation

Strengthen—PR. The correct answer choice is (E)

This author tells us that 99 out of 100 burglar alarm calls are false alarms, with the false alarms wasting an average of 45 minutes of police time each. The author points out that this takes police away from legitimate calls (this implies that something must be done about this problem). The author then points out that “however, burglar alarms…deter burglaries.” This is a concession to the fact that burglar alarms do serve an important purpose. The author closes with what is presented as the “only acceptable solution”: a fine imposed for false alarms.

This question stem requires that we find the principle that would strengthen the case for false alarm fines.

Answer choice (A): The stimulus is not about disproportionate use of police service, but about wasting police time with false alarms (as is done more often by businesses and the wealthy). Since this choice would not strengthen the case for false alarm fines, this answer choice cannot be correct.

Answer choice (B): The stimulus is not about “special services,” but about overuse of standard police services by false alarms. Regardless, this principle would not strengthen the author’s argument for fines, so this answer choice is incorrect.

Answer choice (C): Like incorrect answer choice (B) above, this choice misses the mark, because the author’s argument is not about improving police services, but about what to do regarding the currently available services that are wasted with false alarms.

Answer choice (D): This answer choice goes too far, as this principle would require anyone in need of police services to reimburse for their costs. The author is arguing to fine not everyone who benefits from police services, but rather specifically those responsible for the waste associated with false alarms.

Answer choice (E): This is the correct answer choice. This principle, if applied, would firmly establish that those whose alarms are set all falsely waste police resources and should thus compensate the public for the waste.
 Dungenext
  • Posts: 1
  • Joined: Dec 20, 2016
|
#31652
I understand that choice E is the best answer available, but I'm having a hard time wrapping my head how this completely justifies the conclusion. Two aspects of the stimulus -- mention of car alarms and the phrase "the only" that introduces the conclusion -- cause confusion.

If answer E is chosen, then should not false car alarms result in compensation to the public? Coupled with the phrase "the only" which I understand implies only burglar alarms (no car alarms), I don't see how the answer sufficiently justifies the conclusion. :-?
 Kristina Moen
PowerScore Staff
  • PowerScore Staff
  • Posts: 230
  • Joined: Nov 17, 2016
|
#31669
Hi dungenext,

The conclusion is about burglar alarm systems. The author makes the conclusion that burglar alarm system owners should be fined for false alarms, but the author never says whether car alarm owners should be fined or not fined. In fact, the author may think car alarms should be banned altogether, but we don't know. Because burglar alarms are effective in deterring burglaries, it may be a reason why the author suggests a fine system, rather than an outright ban. But all we know is that the author concludes that burglar alarm systems should be fined for false alarms.
 nikaar
  • Posts: 6
  • Joined: Apr 13, 2018
|
#47619
Hi,

I chose the correct answer, E, for this question but it took me around 2:30 minutes to get to this answer. Any suggestion on how I can get to the answer more quickly when both the stimulus and the answer choices are relatively long?

Thanks!
 Jennifer Janowsky
PowerScore Staff
  • PowerScore Staff
  • Posts: 90
  • Joined: Aug 20, 2017
|
#47760
Nikaar,

Great question! This is a huge stimulus, and the answer choices aren't much smaller. :roll: The best thing to do in situations like these with time consuming LR problems is to practice prephrasing. For example, this is a fast way to solve this problem without having to weigh the pros and cons of each answer choice:

First, when reading the problem, you can already see something fishy in this statement: "so the only acceptable solution is to fine burglar alarm system owners the cost of 45 minutes of police time for each false alarm their systems generate." This doesn't seem to present enough evidence. Alarm bells might go off in your head, but you can just set it aside as you move on.

Next, when you read the question stem, try to figure out what type of problem it presents rather than trying to untangle the jumble of words. This stem is long and complicated, but all it's doing is asking you to Strengthen the stimulus.

Therefore it's a strengthen question, and you're already ahead for the next step because you've already found the arguments weakness. Now all you have to do is look for the answer choice that best amends the weakness you've already spotted!

This might not lead you to the right answer immediately, but it should at least allow you to narrow down more efficiently without getting tripped up in the wordiness. For example, in this situation it helped me eliminate everything but (D) and (E), and let me waste a bit less time pondering the other choices.

I hope that helps you out a little bit, thanks for your question!
 flowskiferda
  • Posts: 30
  • Joined: Sep 19, 2020
|
#79090
I'm having trouble understanding why C is wrong. Doesn't it also need to be true for the argument to work? By stating that they should penalize false alarms (rather than banning all burglar alarm system outright), the argument implies that these burglar alarm systems are important enough to keep due to their crime deterring properties.
Is this wrong because "effective in deterring burglaries" cannot be extrapolated to "reducing crime throughout the entire area"?
 Jeremy Press
PowerScore Staff
  • PowerScore Staff
  • Posts: 1000
  • Joined: Jun 12, 2017
|
#79141
Hi flow,

You're right to say that we cannot know whether the exception stated in the rule in answer choice C would apply, because we don't know whether police responding to burglar alarms "reduces the crime level throughout the entire area served."

Another main problem with answer choice C is that it speaks to a factual situation in which police are "improving service to one segment of the community." But, the stimulus does not give us facts to assume that, by responding to so many false burglar alarms, the police are "improving" the service that alarm owners receive (relative to what they had been receiving before the alarms were used). If anything, the stimulus only sketches out a factual scenario in which service is being wasted on the alarm owners.

Even more importantly, the conclusion of the stimulus argument (which is what we want the rule to strengthen) is not about police. Rather, the conclusion is about what burglar alarm owners should have to do. The rule in answer choice C is about whether police are justified (or not justified) in what they do. The conclusion of the stimulus isn't taking any position on whether the police are acting in a way that's justified. Focus on an answer that speaks to the alarm owners, e.g. answer choice E which talks about what such owners should do ("should compensate the public").

I hope this helps!

Get the most out of your LSAT Prep Plus subscription.

Analyze and track your performance with our Testing and Analytics Package.